Joint density function of $X$ and $X-Y$, where $X, Ysim U(-1,1)$












0














Let $X$ and $Y$ be independent random variables following $U(-1,1)$. Find the joint CDF of $U=X-Y$ and $V=X$.



I found the Jacobian of the transformation to be equal to $1$, and $f_Xf_Y=frac{1}{4}$. I'm confused regarding the final solution.



Is $displaystyle P(Uleq s,Vleq t) = int_{-1}^{t}int_{-2}^{s}frac{1}{8}du dv$? Somehow this seems wrong to me.










share|cite|improve this question
























  • I guess you meant $P()$ instead of $F()$, no? Also, you surely need to specify the range of $s,t$, no?
    – leonbloy
    Nov 24 at 2:14










  • Yes. I'm trying to edit it. $-2<s<2$ and $-1<t<1$.
    – FreeSid91
    Nov 24 at 2:26
















0














Let $X$ and $Y$ be independent random variables following $U(-1,1)$. Find the joint CDF of $U=X-Y$ and $V=X$.



I found the Jacobian of the transformation to be equal to $1$, and $f_Xf_Y=frac{1}{4}$. I'm confused regarding the final solution.



Is $displaystyle P(Uleq s,Vleq t) = int_{-1}^{t}int_{-2}^{s}frac{1}{8}du dv$? Somehow this seems wrong to me.










share|cite|improve this question
























  • I guess you meant $P()$ instead of $F()$, no? Also, you surely need to specify the range of $s,t$, no?
    – leonbloy
    Nov 24 at 2:14










  • Yes. I'm trying to edit it. $-2<s<2$ and $-1<t<1$.
    – FreeSid91
    Nov 24 at 2:26














0












0








0







Let $X$ and $Y$ be independent random variables following $U(-1,1)$. Find the joint CDF of $U=X-Y$ and $V=X$.



I found the Jacobian of the transformation to be equal to $1$, and $f_Xf_Y=frac{1}{4}$. I'm confused regarding the final solution.



Is $displaystyle P(Uleq s,Vleq t) = int_{-1}^{t}int_{-2}^{s}frac{1}{8}du dv$? Somehow this seems wrong to me.










share|cite|improve this question















Let $X$ and $Y$ be independent random variables following $U(-1,1)$. Find the joint CDF of $U=X-Y$ and $V=X$.



I found the Jacobian of the transformation to be equal to $1$, and $f_Xf_Y=frac{1}{4}$. I'm confused regarding the final solution.



Is $displaystyle P(Uleq s,Vleq t) = int_{-1}^{t}int_{-2}^{s}frac{1}{8}du dv$? Somehow this seems wrong to me.







probability density-function






share|cite|improve this question















share|cite|improve this question













share|cite|improve this question




share|cite|improve this question








edited Nov 24 at 2:52









Rócherz

2,7262721




2,7262721










asked Nov 24 at 2:07









FreeSid91

1




1












  • I guess you meant $P()$ instead of $F()$, no? Also, you surely need to specify the range of $s,t$, no?
    – leonbloy
    Nov 24 at 2:14










  • Yes. I'm trying to edit it. $-2<s<2$ and $-1<t<1$.
    – FreeSid91
    Nov 24 at 2:26


















  • I guess you meant $P()$ instead of $F()$, no? Also, you surely need to specify the range of $s,t$, no?
    – leonbloy
    Nov 24 at 2:14










  • Yes. I'm trying to edit it. $-2<s<2$ and $-1<t<1$.
    – FreeSid91
    Nov 24 at 2:26
















I guess you meant $P()$ instead of $F()$, no? Also, you surely need to specify the range of $s,t$, no?
– leonbloy
Nov 24 at 2:14




I guess you meant $P()$ instead of $F()$, no? Also, you surely need to specify the range of $s,t$, no?
– leonbloy
Nov 24 at 2:14












Yes. I'm trying to edit it. $-2<s<2$ and $-1<t<1$.
– FreeSid91
Nov 24 at 2:26




Yes. I'm trying to edit it. $-2<s<2$ and $-1<t<1$.
– FreeSid91
Nov 24 at 2:26










1 Answer
1






active

oldest

votes


















0














If you are using the transformation formula, you should have got, for the joint density



$$f_{U,V}(u,v)=frac{f_{X,Y}(x,y)}{left |frac{partial(U,V) }{partial(X,Y)}right|}=frac{1}{4}$$



but this is not the end of the story, you need to get the support of the transformed variables. To write $-2<U<2$ and $-1<V<1$ is not totally right: both inequalities are true, but they don't give you the support, because (say) you cannot have simultaneously $V=0.9$ and $U=1.9$.



(You can also guess that something is wrong in that the integral of the density over the support must be one).



The correct way is to note that if we allow for $V=X$ its full range $-1<V<1$ then we must put that dependence into the other variable: $U=Y-X=Y-V$ , hence the range of $U$ is $(-1-V, 1-V)$



Then the support is $-1<V<1$ and $-1-V< U <1-V$ which corresponds to a parallelogram.



(Notice BTW that two variables with uniform joint density over a -straight- rectangular support are independent - which is the case for $X,Y$, but it's not -it should not- for $U,V$ )



Sanity check:



$$ int f_{U,V}= int_{-1}^1 int_{-1-V}^{1-V} frac{1}{4} dU dV= frac{1}{4} int_{-1}^1 2 dV= 1$$






share|cite|improve this answer























  • Doesn't the Jacobian have to multiply $f_{X,Y}(x,y)$ instead of divide it? (here it doesn't matter much because $|J| = 1$).
    – Thomas Bladt
    Nov 24 at 3:05










  • @leonbloy Thanks! I was getting confused as I was only changing the lower bound of $U$ instead of both.
    – FreeSid91
    Nov 24 at 3:50












  • @ThomasBladt It depens on which derivative we are taking. I clarified the notation
    – leonbloy
    Nov 24 at 11:20











Your Answer





StackExchange.ifUsing("editor", function () {
return StackExchange.using("mathjaxEditing", function () {
StackExchange.MarkdownEditor.creationCallbacks.add(function (editor, postfix) {
StackExchange.mathjaxEditing.prepareWmdForMathJax(editor, postfix, [["$", "$"], ["\\(","\\)"]]);
});
});
}, "mathjax-editing");

StackExchange.ready(function() {
var channelOptions = {
tags: "".split(" "),
id: "69"
};
initTagRenderer("".split(" "), "".split(" "), channelOptions);

StackExchange.using("externalEditor", function() {
// Have to fire editor after snippets, if snippets enabled
if (StackExchange.settings.snippets.snippetsEnabled) {
StackExchange.using("snippets", function() {
createEditor();
});
}
else {
createEditor();
}
});

function createEditor() {
StackExchange.prepareEditor({
heartbeatType: 'answer',
autoActivateHeartbeat: false,
convertImagesToLinks: true,
noModals: true,
showLowRepImageUploadWarning: true,
reputationToPostImages: 10,
bindNavPrevention: true,
postfix: "",
imageUploader: {
brandingHtml: "Powered by u003ca class="icon-imgur-white" href="https://imgur.com/"u003eu003c/au003e",
contentPolicyHtml: "User contributions licensed under u003ca href="https://creativecommons.org/licenses/by-sa/3.0/"u003ecc by-sa 3.0 with attribution requiredu003c/au003e u003ca href="https://stackoverflow.com/legal/content-policy"u003e(content policy)u003c/au003e",
allowUrls: true
},
noCode: true, onDemand: true,
discardSelector: ".discard-answer"
,immediatelyShowMarkdownHelp:true
});


}
});














draft saved

draft discarded


















StackExchange.ready(
function () {
StackExchange.openid.initPostLogin('.new-post-login', 'https%3a%2f%2fmath.stackexchange.com%2fquestions%2f3011095%2fjoint-density-function-of-x-and-x-y-where-x-y-sim-u-1-1%23new-answer', 'question_page');
}
);

Post as a guest















Required, but never shown

























1 Answer
1






active

oldest

votes








1 Answer
1






active

oldest

votes









active

oldest

votes






active

oldest

votes









0














If you are using the transformation formula, you should have got, for the joint density



$$f_{U,V}(u,v)=frac{f_{X,Y}(x,y)}{left |frac{partial(U,V) }{partial(X,Y)}right|}=frac{1}{4}$$



but this is not the end of the story, you need to get the support of the transformed variables. To write $-2<U<2$ and $-1<V<1$ is not totally right: both inequalities are true, but they don't give you the support, because (say) you cannot have simultaneously $V=0.9$ and $U=1.9$.



(You can also guess that something is wrong in that the integral of the density over the support must be one).



The correct way is to note that if we allow for $V=X$ its full range $-1<V<1$ then we must put that dependence into the other variable: $U=Y-X=Y-V$ , hence the range of $U$ is $(-1-V, 1-V)$



Then the support is $-1<V<1$ and $-1-V< U <1-V$ which corresponds to a parallelogram.



(Notice BTW that two variables with uniform joint density over a -straight- rectangular support are independent - which is the case for $X,Y$, but it's not -it should not- for $U,V$ )



Sanity check:



$$ int f_{U,V}= int_{-1}^1 int_{-1-V}^{1-V} frac{1}{4} dU dV= frac{1}{4} int_{-1}^1 2 dV= 1$$






share|cite|improve this answer























  • Doesn't the Jacobian have to multiply $f_{X,Y}(x,y)$ instead of divide it? (here it doesn't matter much because $|J| = 1$).
    – Thomas Bladt
    Nov 24 at 3:05










  • @leonbloy Thanks! I was getting confused as I was only changing the lower bound of $U$ instead of both.
    – FreeSid91
    Nov 24 at 3:50












  • @ThomasBladt It depens on which derivative we are taking. I clarified the notation
    – leonbloy
    Nov 24 at 11:20
















0














If you are using the transformation formula, you should have got, for the joint density



$$f_{U,V}(u,v)=frac{f_{X,Y}(x,y)}{left |frac{partial(U,V) }{partial(X,Y)}right|}=frac{1}{4}$$



but this is not the end of the story, you need to get the support of the transformed variables. To write $-2<U<2$ and $-1<V<1$ is not totally right: both inequalities are true, but they don't give you the support, because (say) you cannot have simultaneously $V=0.9$ and $U=1.9$.



(You can also guess that something is wrong in that the integral of the density over the support must be one).



The correct way is to note that if we allow for $V=X$ its full range $-1<V<1$ then we must put that dependence into the other variable: $U=Y-X=Y-V$ , hence the range of $U$ is $(-1-V, 1-V)$



Then the support is $-1<V<1$ and $-1-V< U <1-V$ which corresponds to a parallelogram.



(Notice BTW that two variables with uniform joint density over a -straight- rectangular support are independent - which is the case for $X,Y$, but it's not -it should not- for $U,V$ )



Sanity check:



$$ int f_{U,V}= int_{-1}^1 int_{-1-V}^{1-V} frac{1}{4} dU dV= frac{1}{4} int_{-1}^1 2 dV= 1$$






share|cite|improve this answer























  • Doesn't the Jacobian have to multiply $f_{X,Y}(x,y)$ instead of divide it? (here it doesn't matter much because $|J| = 1$).
    – Thomas Bladt
    Nov 24 at 3:05










  • @leonbloy Thanks! I was getting confused as I was only changing the lower bound of $U$ instead of both.
    – FreeSid91
    Nov 24 at 3:50












  • @ThomasBladt It depens on which derivative we are taking. I clarified the notation
    – leonbloy
    Nov 24 at 11:20














0












0








0






If you are using the transformation formula, you should have got, for the joint density



$$f_{U,V}(u,v)=frac{f_{X,Y}(x,y)}{left |frac{partial(U,V) }{partial(X,Y)}right|}=frac{1}{4}$$



but this is not the end of the story, you need to get the support of the transformed variables. To write $-2<U<2$ and $-1<V<1$ is not totally right: both inequalities are true, but they don't give you the support, because (say) you cannot have simultaneously $V=0.9$ and $U=1.9$.



(You can also guess that something is wrong in that the integral of the density over the support must be one).



The correct way is to note that if we allow for $V=X$ its full range $-1<V<1$ then we must put that dependence into the other variable: $U=Y-X=Y-V$ , hence the range of $U$ is $(-1-V, 1-V)$



Then the support is $-1<V<1$ and $-1-V< U <1-V$ which corresponds to a parallelogram.



(Notice BTW that two variables with uniform joint density over a -straight- rectangular support are independent - which is the case for $X,Y$, but it's not -it should not- for $U,V$ )



Sanity check:



$$ int f_{U,V}= int_{-1}^1 int_{-1-V}^{1-V} frac{1}{4} dU dV= frac{1}{4} int_{-1}^1 2 dV= 1$$






share|cite|improve this answer














If you are using the transformation formula, you should have got, for the joint density



$$f_{U,V}(u,v)=frac{f_{X,Y}(x,y)}{left |frac{partial(U,V) }{partial(X,Y)}right|}=frac{1}{4}$$



but this is not the end of the story, you need to get the support of the transformed variables. To write $-2<U<2$ and $-1<V<1$ is not totally right: both inequalities are true, but they don't give you the support, because (say) you cannot have simultaneously $V=0.9$ and $U=1.9$.



(You can also guess that something is wrong in that the integral of the density over the support must be one).



The correct way is to note that if we allow for $V=X$ its full range $-1<V<1$ then we must put that dependence into the other variable: $U=Y-X=Y-V$ , hence the range of $U$ is $(-1-V, 1-V)$



Then the support is $-1<V<1$ and $-1-V< U <1-V$ which corresponds to a parallelogram.



(Notice BTW that two variables with uniform joint density over a -straight- rectangular support are independent - which is the case for $X,Y$, but it's not -it should not- for $U,V$ )



Sanity check:



$$ int f_{U,V}= int_{-1}^1 int_{-1-V}^{1-V} frac{1}{4} dU dV= frac{1}{4} int_{-1}^1 2 dV= 1$$







share|cite|improve this answer














share|cite|improve this answer



share|cite|improve this answer








edited Nov 24 at 11:19

























answered Nov 24 at 2:47









leonbloy

40.2k645107




40.2k645107












  • Doesn't the Jacobian have to multiply $f_{X,Y}(x,y)$ instead of divide it? (here it doesn't matter much because $|J| = 1$).
    – Thomas Bladt
    Nov 24 at 3:05










  • @leonbloy Thanks! I was getting confused as I was only changing the lower bound of $U$ instead of both.
    – FreeSid91
    Nov 24 at 3:50












  • @ThomasBladt It depens on which derivative we are taking. I clarified the notation
    – leonbloy
    Nov 24 at 11:20


















  • Doesn't the Jacobian have to multiply $f_{X,Y}(x,y)$ instead of divide it? (here it doesn't matter much because $|J| = 1$).
    – Thomas Bladt
    Nov 24 at 3:05










  • @leonbloy Thanks! I was getting confused as I was only changing the lower bound of $U$ instead of both.
    – FreeSid91
    Nov 24 at 3:50












  • @ThomasBladt It depens on which derivative we are taking. I clarified the notation
    – leonbloy
    Nov 24 at 11:20
















Doesn't the Jacobian have to multiply $f_{X,Y}(x,y)$ instead of divide it? (here it doesn't matter much because $|J| = 1$).
– Thomas Bladt
Nov 24 at 3:05




Doesn't the Jacobian have to multiply $f_{X,Y}(x,y)$ instead of divide it? (here it doesn't matter much because $|J| = 1$).
– Thomas Bladt
Nov 24 at 3:05












@leonbloy Thanks! I was getting confused as I was only changing the lower bound of $U$ instead of both.
– FreeSid91
Nov 24 at 3:50






@leonbloy Thanks! I was getting confused as I was only changing the lower bound of $U$ instead of both.
– FreeSid91
Nov 24 at 3:50














@ThomasBladt It depens on which derivative we are taking. I clarified the notation
– leonbloy
Nov 24 at 11:20




@ThomasBladt It depens on which derivative we are taking. I clarified the notation
– leonbloy
Nov 24 at 11:20


















draft saved

draft discarded




















































Thanks for contributing an answer to Mathematics Stack Exchange!


  • Please be sure to answer the question. Provide details and share your research!

But avoid



  • Asking for help, clarification, or responding to other answers.

  • Making statements based on opinion; back them up with references or personal experience.


Use MathJax to format equations. MathJax reference.


To learn more, see our tips on writing great answers.





Some of your past answers have not been well-received, and you're in danger of being blocked from answering.


Please pay close attention to the following guidance:


  • Please be sure to answer the question. Provide details and share your research!

But avoid



  • Asking for help, clarification, or responding to other answers.

  • Making statements based on opinion; back them up with references or personal experience.


To learn more, see our tips on writing great answers.




draft saved


draft discarded














StackExchange.ready(
function () {
StackExchange.openid.initPostLogin('.new-post-login', 'https%3a%2f%2fmath.stackexchange.com%2fquestions%2f3011095%2fjoint-density-function-of-x-and-x-y-where-x-y-sim-u-1-1%23new-answer', 'question_page');
}
);

Post as a guest















Required, but never shown





















































Required, but never shown














Required, but never shown












Required, but never shown







Required, but never shown

































Required, but never shown














Required, but never shown












Required, but never shown







Required, but never shown







Popular posts from this blog

Quarter-circle Tiles

build a pushdown automaton that recognizes the reverse language of a given pushdown automaton?

Mont Emei